Question on the limit of a sequence if the sum of the sequence converges












2












$begingroup$


Suppose I have a sequence ${a_i}_{i=1}^{infty}$. I know that $a_i geq 0$ for all $i$ and $sum_{i=0}^{infty}a_i < infty$. Can I say that



$$ lim_{i to infty} a_i = 0$$



Intuitively, I think this is true. If the limit tends to a positive number, the sum would explode as well. However, I'm having a hard time proving it formally. Can I get a hint?










share|cite|improve this question









$endgroup$

















    2












    $begingroup$


    Suppose I have a sequence ${a_i}_{i=1}^{infty}$. I know that $a_i geq 0$ for all $i$ and $sum_{i=0}^{infty}a_i < infty$. Can I say that



    $$ lim_{i to infty} a_i = 0$$



    Intuitively, I think this is true. If the limit tends to a positive number, the sum would explode as well. However, I'm having a hard time proving it formally. Can I get a hint?










    share|cite|improve this question









    $endgroup$















      2












      2








      2





      $begingroup$


      Suppose I have a sequence ${a_i}_{i=1}^{infty}$. I know that $a_i geq 0$ for all $i$ and $sum_{i=0}^{infty}a_i < infty$. Can I say that



      $$ lim_{i to infty} a_i = 0$$



      Intuitively, I think this is true. If the limit tends to a positive number, the sum would explode as well. However, I'm having a hard time proving it formally. Can I get a hint?










      share|cite|improve this question









      $endgroup$




      Suppose I have a sequence ${a_i}_{i=1}^{infty}$. I know that $a_i geq 0$ for all $i$ and $sum_{i=0}^{infty}a_i < infty$. Can I say that



      $$ lim_{i to infty} a_i = 0$$



      Intuitively, I think this is true. If the limit tends to a positive number, the sum would explode as well. However, I'm having a hard time proving it formally. Can I get a hint?







      sequences-and-series limits summation






      share|cite|improve this question













      share|cite|improve this question











      share|cite|improve this question




      share|cite|improve this question










      asked 6 hours ago









      user1691278user1691278

      47139




      47139






















          3 Answers
          3






          active

          oldest

          votes


















          5












          $begingroup$

          We have
          $$sum_{i=0}^n a_ito S ,qquad sum_{i=0}^{n-1} a_ito S$$
          as $ntoinfty$, so
          $$a_n=Bigl(sum_{i=0}^n a_iBigr)-Bigl(sum_{i=0}^{n-1} a_iBigr)to S-S=0 .$$






          share|cite|improve this answer









          $endgroup$













          • $begingroup$
            Great proof. Is $a_i > 0$ even necessary?
            $endgroup$
            – user1691278
            6 hours ago












          • $begingroup$
            No.$!,!,!,$
            $endgroup$
            – David
            6 hours ago



















          1












          $begingroup$

          I'm not quite sure if the sequence starts at index $0$ or $1$ as your question uses $1$ as the sequence start but the summation starts at $0$. As it's not important, I will assume it's $0$. Note that



          $$sum_{i , = , 0}^{infty} a_i lt infty tag{1}label{eq1}$$



          usually means it converges to a limit. However, consider the somewhat looser restriction of there just simply existing a real supremum, call it $S$, so that



          $$sum_{i , = , 0}^{n} a_i le S ; forall ; n ge 0 tag{2}label{eq2}$$



          As trying to show the limit of $a_i$ is $0$, I will just let the $- 0$ be implied. Also, it's given that $a_i ge 0$, so the partial sums are non-decreasing, and there are no non-negative values so I don't need to use absolute values in the following. For proving that



          $$lim_{i , to , infty} a_i = 0 tag{3}label{eq3}$$



          consider that for any $epsilon gt 0$ there must be a finite number of integers for which $a_n ge epsilon$ as, otherwise, the sum of them would be unbounded. Thus, there must be either none or a maximum index, but in either choose an integer $n_0$ to be greater than any such value and, thus, $a_n lt epsilon$ for all $n gt n_0$, which confirms that eqref{eq3} is true. I won't show it here, but the infinite sum has a limit as well of $S$.



          Note that assuming just a supremum for the sums without assuming that $a_i ge 0$ won't suffice to show the limit as, for example, $a_i = left(-1right)^i$ would cause the partial sums to alternate between $1$ and $0$, thus allowing a supremum of $1$, but with $a_i$ not converging to $0$.



          Note that requiring the sum of $a_i$ to have a finite limit means that, as David indicated in his answer's comment, also requiring the $a_i$ to be non-negative is unnecessary. However, if the requirement of eqref{eq1} includes the possibility of the limit being $-infty$, then some appropriate restrictions on $a_i$ would be required.






          share|cite|improve this answer











          $endgroup$





















            0












            $begingroup$

            Informal outline for a more formal proof:



            Let $s_n$ be the sum of the first $n$ terms and let the limit of the series be $m$.



            Convergence means that for as small an $epsilon>0$ as we like and large enough $N$, $n>N$ guarantees $|m-s_n|<epsilon$.



            But this goes awry if ${a_n}$ doesn't converge to $0$, because when $2epsilon<|a_{n+1}|$ it's not possible to have both $|m-s_n|<epsilon$ and $|m-s_{n+1}|<epsilon$.



            (And use the definition of ${a_n}$ not converging to show that this situation must arise.)






            share|cite|improve this answer











            $endgroup$













              Your Answer





              StackExchange.ifUsing("editor", function () {
              return StackExchange.using("mathjaxEditing", function () {
              StackExchange.MarkdownEditor.creationCallbacks.add(function (editor, postfix) {
              StackExchange.mathjaxEditing.prepareWmdForMathJax(editor, postfix, [["$", "$"], ["\\(","\\)"]]);
              });
              });
              }, "mathjax-editing");

              StackExchange.ready(function() {
              var channelOptions = {
              tags: "".split(" "),
              id: "69"
              };
              initTagRenderer("".split(" "), "".split(" "), channelOptions);

              StackExchange.using("externalEditor", function() {
              // Have to fire editor after snippets, if snippets enabled
              if (StackExchange.settings.snippets.snippetsEnabled) {
              StackExchange.using("snippets", function() {
              createEditor();
              });
              }
              else {
              createEditor();
              }
              });

              function createEditor() {
              StackExchange.prepareEditor({
              heartbeatType: 'answer',
              autoActivateHeartbeat: false,
              convertImagesToLinks: true,
              noModals: true,
              showLowRepImageUploadWarning: true,
              reputationToPostImages: 10,
              bindNavPrevention: true,
              postfix: "",
              imageUploader: {
              brandingHtml: "Powered by u003ca class="icon-imgur-white" href="https://imgur.com/"u003eu003c/au003e",
              contentPolicyHtml: "User contributions licensed under u003ca href="https://creativecommons.org/licenses/by-sa/3.0/"u003ecc by-sa 3.0 with attribution requiredu003c/au003e u003ca href="https://stackoverflow.com/legal/content-policy"u003e(content policy)u003c/au003e",
              allowUrls: true
              },
              noCode: true, onDemand: true,
              discardSelector: ".discard-answer"
              ,immediatelyShowMarkdownHelp:true
              });


              }
              });














              draft saved

              draft discarded


















              StackExchange.ready(
              function () {
              StackExchange.openid.initPostLogin('.new-post-login', 'https%3a%2f%2fmath.stackexchange.com%2fquestions%2f3076418%2fquestion-on-the-limit-of-a-sequence-if-the-sum-of-the-sequence-converges%23new-answer', 'question_page');
              }
              );

              Post as a guest















              Required, but never shown

























              3 Answers
              3






              active

              oldest

              votes








              3 Answers
              3






              active

              oldest

              votes









              active

              oldest

              votes






              active

              oldest

              votes









              5












              $begingroup$

              We have
              $$sum_{i=0}^n a_ito S ,qquad sum_{i=0}^{n-1} a_ito S$$
              as $ntoinfty$, so
              $$a_n=Bigl(sum_{i=0}^n a_iBigr)-Bigl(sum_{i=0}^{n-1} a_iBigr)to S-S=0 .$$






              share|cite|improve this answer









              $endgroup$













              • $begingroup$
                Great proof. Is $a_i > 0$ even necessary?
                $endgroup$
                – user1691278
                6 hours ago












              • $begingroup$
                No.$!,!,!,$
                $endgroup$
                – David
                6 hours ago
















              5












              $begingroup$

              We have
              $$sum_{i=0}^n a_ito S ,qquad sum_{i=0}^{n-1} a_ito S$$
              as $ntoinfty$, so
              $$a_n=Bigl(sum_{i=0}^n a_iBigr)-Bigl(sum_{i=0}^{n-1} a_iBigr)to S-S=0 .$$






              share|cite|improve this answer









              $endgroup$













              • $begingroup$
                Great proof. Is $a_i > 0$ even necessary?
                $endgroup$
                – user1691278
                6 hours ago












              • $begingroup$
                No.$!,!,!,$
                $endgroup$
                – David
                6 hours ago














              5












              5








              5





              $begingroup$

              We have
              $$sum_{i=0}^n a_ito S ,qquad sum_{i=0}^{n-1} a_ito S$$
              as $ntoinfty$, so
              $$a_n=Bigl(sum_{i=0}^n a_iBigr)-Bigl(sum_{i=0}^{n-1} a_iBigr)to S-S=0 .$$






              share|cite|improve this answer









              $endgroup$



              We have
              $$sum_{i=0}^n a_ito S ,qquad sum_{i=0}^{n-1} a_ito S$$
              as $ntoinfty$, so
              $$a_n=Bigl(sum_{i=0}^n a_iBigr)-Bigl(sum_{i=0}^{n-1} a_iBigr)to S-S=0 .$$







              share|cite|improve this answer












              share|cite|improve this answer



              share|cite|improve this answer










              answered 6 hours ago









              DavidDavid

              68k664126




              68k664126












              • $begingroup$
                Great proof. Is $a_i > 0$ even necessary?
                $endgroup$
                – user1691278
                6 hours ago












              • $begingroup$
                No.$!,!,!,$
                $endgroup$
                – David
                6 hours ago


















              • $begingroup$
                Great proof. Is $a_i > 0$ even necessary?
                $endgroup$
                – user1691278
                6 hours ago












              • $begingroup$
                No.$!,!,!,$
                $endgroup$
                – David
                6 hours ago
















              $begingroup$
              Great proof. Is $a_i > 0$ even necessary?
              $endgroup$
              – user1691278
              6 hours ago






              $begingroup$
              Great proof. Is $a_i > 0$ even necessary?
              $endgroup$
              – user1691278
              6 hours ago














              $begingroup$
              No.$!,!,!,$
              $endgroup$
              – David
              6 hours ago




              $begingroup$
              No.$!,!,!,$
              $endgroup$
              – David
              6 hours ago











              1












              $begingroup$

              I'm not quite sure if the sequence starts at index $0$ or $1$ as your question uses $1$ as the sequence start but the summation starts at $0$. As it's not important, I will assume it's $0$. Note that



              $$sum_{i , = , 0}^{infty} a_i lt infty tag{1}label{eq1}$$



              usually means it converges to a limit. However, consider the somewhat looser restriction of there just simply existing a real supremum, call it $S$, so that



              $$sum_{i , = , 0}^{n} a_i le S ; forall ; n ge 0 tag{2}label{eq2}$$



              As trying to show the limit of $a_i$ is $0$, I will just let the $- 0$ be implied. Also, it's given that $a_i ge 0$, so the partial sums are non-decreasing, and there are no non-negative values so I don't need to use absolute values in the following. For proving that



              $$lim_{i , to , infty} a_i = 0 tag{3}label{eq3}$$



              consider that for any $epsilon gt 0$ there must be a finite number of integers for which $a_n ge epsilon$ as, otherwise, the sum of them would be unbounded. Thus, there must be either none or a maximum index, but in either choose an integer $n_0$ to be greater than any such value and, thus, $a_n lt epsilon$ for all $n gt n_0$, which confirms that eqref{eq3} is true. I won't show it here, but the infinite sum has a limit as well of $S$.



              Note that assuming just a supremum for the sums without assuming that $a_i ge 0$ won't suffice to show the limit as, for example, $a_i = left(-1right)^i$ would cause the partial sums to alternate between $1$ and $0$, thus allowing a supremum of $1$, but with $a_i$ not converging to $0$.



              Note that requiring the sum of $a_i$ to have a finite limit means that, as David indicated in his answer's comment, also requiring the $a_i$ to be non-negative is unnecessary. However, if the requirement of eqref{eq1} includes the possibility of the limit being $-infty$, then some appropriate restrictions on $a_i$ would be required.






              share|cite|improve this answer











              $endgroup$


















                1












                $begingroup$

                I'm not quite sure if the sequence starts at index $0$ or $1$ as your question uses $1$ as the sequence start but the summation starts at $0$. As it's not important, I will assume it's $0$. Note that



                $$sum_{i , = , 0}^{infty} a_i lt infty tag{1}label{eq1}$$



                usually means it converges to a limit. However, consider the somewhat looser restriction of there just simply existing a real supremum, call it $S$, so that



                $$sum_{i , = , 0}^{n} a_i le S ; forall ; n ge 0 tag{2}label{eq2}$$



                As trying to show the limit of $a_i$ is $0$, I will just let the $- 0$ be implied. Also, it's given that $a_i ge 0$, so the partial sums are non-decreasing, and there are no non-negative values so I don't need to use absolute values in the following. For proving that



                $$lim_{i , to , infty} a_i = 0 tag{3}label{eq3}$$



                consider that for any $epsilon gt 0$ there must be a finite number of integers for which $a_n ge epsilon$ as, otherwise, the sum of them would be unbounded. Thus, there must be either none or a maximum index, but in either choose an integer $n_0$ to be greater than any such value and, thus, $a_n lt epsilon$ for all $n gt n_0$, which confirms that eqref{eq3} is true. I won't show it here, but the infinite sum has a limit as well of $S$.



                Note that assuming just a supremum for the sums without assuming that $a_i ge 0$ won't suffice to show the limit as, for example, $a_i = left(-1right)^i$ would cause the partial sums to alternate between $1$ and $0$, thus allowing a supremum of $1$, but with $a_i$ not converging to $0$.



                Note that requiring the sum of $a_i$ to have a finite limit means that, as David indicated in his answer's comment, also requiring the $a_i$ to be non-negative is unnecessary. However, if the requirement of eqref{eq1} includes the possibility of the limit being $-infty$, then some appropriate restrictions on $a_i$ would be required.






                share|cite|improve this answer











                $endgroup$
















                  1












                  1








                  1





                  $begingroup$

                  I'm not quite sure if the sequence starts at index $0$ or $1$ as your question uses $1$ as the sequence start but the summation starts at $0$. As it's not important, I will assume it's $0$. Note that



                  $$sum_{i , = , 0}^{infty} a_i lt infty tag{1}label{eq1}$$



                  usually means it converges to a limit. However, consider the somewhat looser restriction of there just simply existing a real supremum, call it $S$, so that



                  $$sum_{i , = , 0}^{n} a_i le S ; forall ; n ge 0 tag{2}label{eq2}$$



                  As trying to show the limit of $a_i$ is $0$, I will just let the $- 0$ be implied. Also, it's given that $a_i ge 0$, so the partial sums are non-decreasing, and there are no non-negative values so I don't need to use absolute values in the following. For proving that



                  $$lim_{i , to , infty} a_i = 0 tag{3}label{eq3}$$



                  consider that for any $epsilon gt 0$ there must be a finite number of integers for which $a_n ge epsilon$ as, otherwise, the sum of them would be unbounded. Thus, there must be either none or a maximum index, but in either choose an integer $n_0$ to be greater than any such value and, thus, $a_n lt epsilon$ for all $n gt n_0$, which confirms that eqref{eq3} is true. I won't show it here, but the infinite sum has a limit as well of $S$.



                  Note that assuming just a supremum for the sums without assuming that $a_i ge 0$ won't suffice to show the limit as, for example, $a_i = left(-1right)^i$ would cause the partial sums to alternate between $1$ and $0$, thus allowing a supremum of $1$, but with $a_i$ not converging to $0$.



                  Note that requiring the sum of $a_i$ to have a finite limit means that, as David indicated in his answer's comment, also requiring the $a_i$ to be non-negative is unnecessary. However, if the requirement of eqref{eq1} includes the possibility of the limit being $-infty$, then some appropriate restrictions on $a_i$ would be required.






                  share|cite|improve this answer











                  $endgroup$



                  I'm not quite sure if the sequence starts at index $0$ or $1$ as your question uses $1$ as the sequence start but the summation starts at $0$. As it's not important, I will assume it's $0$. Note that



                  $$sum_{i , = , 0}^{infty} a_i lt infty tag{1}label{eq1}$$



                  usually means it converges to a limit. However, consider the somewhat looser restriction of there just simply existing a real supremum, call it $S$, so that



                  $$sum_{i , = , 0}^{n} a_i le S ; forall ; n ge 0 tag{2}label{eq2}$$



                  As trying to show the limit of $a_i$ is $0$, I will just let the $- 0$ be implied. Also, it's given that $a_i ge 0$, so the partial sums are non-decreasing, and there are no non-negative values so I don't need to use absolute values in the following. For proving that



                  $$lim_{i , to , infty} a_i = 0 tag{3}label{eq3}$$



                  consider that for any $epsilon gt 0$ there must be a finite number of integers for which $a_n ge epsilon$ as, otherwise, the sum of them would be unbounded. Thus, there must be either none or a maximum index, but in either choose an integer $n_0$ to be greater than any such value and, thus, $a_n lt epsilon$ for all $n gt n_0$, which confirms that eqref{eq3} is true. I won't show it here, but the infinite sum has a limit as well of $S$.



                  Note that assuming just a supremum for the sums without assuming that $a_i ge 0$ won't suffice to show the limit as, for example, $a_i = left(-1right)^i$ would cause the partial sums to alternate between $1$ and $0$, thus allowing a supremum of $1$, but with $a_i$ not converging to $0$.



                  Note that requiring the sum of $a_i$ to have a finite limit means that, as David indicated in his answer's comment, also requiring the $a_i$ to be non-negative is unnecessary. However, if the requirement of eqref{eq1} includes the possibility of the limit being $-infty$, then some appropriate restrictions on $a_i$ would be required.







                  share|cite|improve this answer














                  share|cite|improve this answer



                  share|cite|improve this answer








                  edited 3 hours ago

























                  answered 5 hours ago









                  John OmielanJohn Omielan

                  1,28629




                  1,28629























                      0












                      $begingroup$

                      Informal outline for a more formal proof:



                      Let $s_n$ be the sum of the first $n$ terms and let the limit of the series be $m$.



                      Convergence means that for as small an $epsilon>0$ as we like and large enough $N$, $n>N$ guarantees $|m-s_n|<epsilon$.



                      But this goes awry if ${a_n}$ doesn't converge to $0$, because when $2epsilon<|a_{n+1}|$ it's not possible to have both $|m-s_n|<epsilon$ and $|m-s_{n+1}|<epsilon$.



                      (And use the definition of ${a_n}$ not converging to show that this situation must arise.)






                      share|cite|improve this answer











                      $endgroup$


















                        0












                        $begingroup$

                        Informal outline for a more formal proof:



                        Let $s_n$ be the sum of the first $n$ terms and let the limit of the series be $m$.



                        Convergence means that for as small an $epsilon>0$ as we like and large enough $N$, $n>N$ guarantees $|m-s_n|<epsilon$.



                        But this goes awry if ${a_n}$ doesn't converge to $0$, because when $2epsilon<|a_{n+1}|$ it's not possible to have both $|m-s_n|<epsilon$ and $|m-s_{n+1}|<epsilon$.



                        (And use the definition of ${a_n}$ not converging to show that this situation must arise.)






                        share|cite|improve this answer











                        $endgroup$
















                          0












                          0








                          0





                          $begingroup$

                          Informal outline for a more formal proof:



                          Let $s_n$ be the sum of the first $n$ terms and let the limit of the series be $m$.



                          Convergence means that for as small an $epsilon>0$ as we like and large enough $N$, $n>N$ guarantees $|m-s_n|<epsilon$.



                          But this goes awry if ${a_n}$ doesn't converge to $0$, because when $2epsilon<|a_{n+1}|$ it's not possible to have both $|m-s_n|<epsilon$ and $|m-s_{n+1}|<epsilon$.



                          (And use the definition of ${a_n}$ not converging to show that this situation must arise.)






                          share|cite|improve this answer











                          $endgroup$



                          Informal outline for a more formal proof:



                          Let $s_n$ be the sum of the first $n$ terms and let the limit of the series be $m$.



                          Convergence means that for as small an $epsilon>0$ as we like and large enough $N$, $n>N$ guarantees $|m-s_n|<epsilon$.



                          But this goes awry if ${a_n}$ doesn't converge to $0$, because when $2epsilon<|a_{n+1}|$ it's not possible to have both $|m-s_n|<epsilon$ and $|m-s_{n+1}|<epsilon$.



                          (And use the definition of ${a_n}$ not converging to show that this situation must arise.)







                          share|cite|improve this answer














                          share|cite|improve this answer



                          share|cite|improve this answer








                          edited 4 hours ago

























                          answered 5 hours ago









                          timtfjtimtfj

                          1,318318




                          1,318318






























                              draft saved

                              draft discarded




















































                              Thanks for contributing an answer to Mathematics Stack Exchange!


                              • Please be sure to answer the question. Provide details and share your research!

                              But avoid



                              • Asking for help, clarification, or responding to other answers.

                              • Making statements based on opinion; back them up with references or personal experience.


                              Use MathJax to format equations. MathJax reference.


                              To learn more, see our tips on writing great answers.




                              draft saved


                              draft discarded














                              StackExchange.ready(
                              function () {
                              StackExchange.openid.initPostLogin('.new-post-login', 'https%3a%2f%2fmath.stackexchange.com%2fquestions%2f3076418%2fquestion-on-the-limit-of-a-sequence-if-the-sum-of-the-sequence-converges%23new-answer', 'question_page');
                              }
                              );

                              Post as a guest















                              Required, but never shown





















































                              Required, but never shown














                              Required, but never shown












                              Required, but never shown







                              Required, but never shown

































                              Required, but never shown














                              Required, but never shown












                              Required, but never shown







                              Required, but never shown







                              Popular posts from this blog

                              サソリ

                              広島県道265号伴広島線

                              Accessing regular linux commands in Huawei's Dopra Linux